luke and matthew ran a lemond stand on saturday. they agreed that matthew would get 60% persent of the profit because the lemonde stand was his idea. they made a profit of 25. how much money did luke get

Answers

Answer 1
Luke would get 10 since mathew would be getting 15

Related Questions

what is the answer for the question A=B+C

Answers

Answer:

8=4+4

Step-by-step explanation:

6. How many planes contain the given line and point?
a. DB and point A
b. BD and point E
C. AC and point D
d. EB and point C

Answers

Answer:

B. BD and point E

Step-by-step explanation:

BD and point E is the only choice with both of them lying on the plane, the others lie on other planes

What have you learned about geometric relationships?
PLEASE HELP 15 POINTS

Answers

Answer:

that you can define a tangent relationship between a line and an arc. If the adjoining elements change, the tangent relationship is maintained between the elements. Geometric relationships control how a sketch changes when edits are made.

Step-by-step explanation:

i took this i guess and it was gud

Perpendicular to y=-2x+1, but passes through (-4,4)

Answers

y=1/2x+6 is the new equation
Remember, the perpendicular slope is always the opposite reciprocal so the slope of this would be 1/2. Then, plug in the new slope along with the point in slope intercept form to solve for b.

The perimeter of a rectangle measures 36 inches. If the width is 5 times the length, what is the width of the rectangle?

Answers

Answer:

the width of the rectangle is 15 inches ...

and the length is 3 inches

Answer:

180.

Step-by-step explanation:

to find this, you multiply 36 by 5, which gets you 180.

keyword: times

Write a phrase for the expression 3(5−2m).

Answers

Answer:

three times 5 is fifteen and three times two is six m

Step-by-step explanation:

15 points and brainliest if right!!

Answers

Answer:

y + 5x = -7

Step-by-step explanation:

Hello there?

y + 3 = -5(x + 2)

LHS

Open the brackets

= -5x - 10

RHS remains

Combine the two:

y + 3 = -5 - 10

Collecting the like terms and taking the value of x to the LHS

=> y + 5x = - 7

I hope this helps. Have a nice studies

Which of the following describes the transformation from Figure 1 to Figure 2? On a coordinate plane, figure A B C D E has points (negative 3, 5), (negative 2, 5), (negative 1, 4), (negative 2, 3), (negative 5, 3). Figure A prime B prime C prime D prime E prime has points (2, 2), (3, 2), (4, 1), (3, 0), (0, 0). CLEAR CHECK translation 2 units to the right and 3 units down translation 3 units to the left and 2 units up translation 5 units to the right and 3 units down translation 5 units to the left and 3 units up

Answers

Answer:

a

Step-by-step explanation:

The transformation from Figure 1 to Figure 2 is:

The transformation of 5 units to the right and 3 units down.

Option C is the correct answer.

What is translation?

It is the movement of the shape in the left, right, up, and down directions.

The translated shape will have the same shape and shape.

There is a positive value when translated to the right and up.

There is a negative value when translated to the left and down.

We have,

A B C D E has points (-3, 5), (-2, 5), (-1, 4), (-2, 3), and (-5, 3).

A' B' C' D' E' has points (2, 2), (3, 2), (4, 1), (3, 0), and (0, 0).

Now,

A = (-3 + 5, 5 - 3) to A' = (2, 2)

B = (-2 + 5, 5 - 3) to B' = (3, 2)

C = (-1 + 5, 4 - 3) to C' = (4, 1)

D = (-2 + 5, 3 - 3) to A' = (3, 0)

E = (-5 + 5, 3 - 3) to E' = (0, 0)

We see that,

There is a translation of 5 units to the right and  3 units to the down.

Thus,

The transformation of 5 units to the right and 3 units down.

Learn more about translation here:

https://brainly.com/question/12463306

#SPJ1

equation of the line that passes through the points (7, 6) and (-2, -3)?

Answers

Answer:

y=x-1

Step-by-step explanation:

Hi there!

We want to find the equation of the line that passes through the points (7, 6) and (-2, -3)

There are 3 ways to write the equation of the line:

Slope-intercept form, which is y=mx+b, where m is the slope and b is the y intercept Point-slope form, which is [tex]y-y_1=m(x-x_1)[/tex], where m is the slope and [tex](x_1, y_1)[/tex] is a pointStandard form, which is ax+by=c, where a, b, and c are free integer coefficients, but a and b cannot be 0, and a cannot be negative

The most common (and usually, the easiest way) would be slope-intercept form, so let's write it that way

First, we'll need to find the slope of the line

The slope can be found using the formula [tex]\frac{y_2-y_1}{x_2-x_1}[/tex], where [tex](x_1, y_1)[/tex] and [tex](x_2, y_2)[/tex] are points

We have two points, which is needed, but let's label their values in order to avoid any confusion:

[tex]x_1= 7\\y_1=6\\x_2=-2\\y_2=-3[/tex]

Now substitute these values into the formula to find the slope (m):

m=[tex]\frac{y_2-y_1}{x_2-x_1}[/tex]

m=[tex]\frac{-3-6}{-2-7}[/tex]

Subtract the numbers

m=[tex]\frac{-9}{-9}[/tex]

Divide

m=1

The slope of the line is 1

So far, we can write the equation of the line as this:

y=1x+b, or y=x+b

We'll need to find b

As the equation passes through both (7,6) and (-2, -3), we can use either one of them to solve for b

Taking (7, 6) for instance, substitute 7 as x and 6 as y:

6=1(7)+b

Multiply

6=7+b

Subtract 7 from both sides

-1=b

Now substitute -1 as b:

y=x-1

Hope this helps!

1 7/8 divided by 2 2/5 if give good response = brainlist

Answers

Answer:

75/96 or in its simplest form, 25/32

Step-by-step explanation:

1. Convert both fractions into improper fractions

1 7/8 becomes 15/8 (1 × 8 = 8, 8 + 7 = 15)2 2/5 becomes 12/5 (2 × 5 = 10, 10 + 2 = 12)

2. Use KFC

Keep the first fraction the sameFlip the second fractionChange the sign from ÷ to ×

15/8 ÷ 12/5 = 15/8 × 5/12

15 × 5 = 758 × 12 = 9675/96 = 25/32 (divide the numerator and denomiator by 3)

Hope this help!

Please help asap! I will give brainliest for correct answer!

Answers

Answer:

0.01miles/day

Step-by-step explanation:

1: Convert 9cm to miles

[tex]9cm*(\frac{1m}{100cm})*(\frac{1km}{1000m})*(\frac{1mile}{1.609km})=0.000055935miles[/tex]

2: Convert 5.5 minutes to days

[tex]5.5minutes*(\frac{1hour}{60minutes})*(\frac{1day}{24hours})=0.003819444 days[/tex]

3. Set miles over days

[tex]\frac{0.000055935miles}{0.003819444days}[/tex]

4: Divide to get the answer

0.014644 miles/day

5: Round to get the final answer

0.01 miles/day

maths lit November question paper paper 1 grade 11 2021?​

Answers

Answer:

can you be more clear i dont get the question (not trying to be mean)

Step-by-step explanation:

Ummmmmmmmmmm???????????

PROBLEM SOLVING A boat is traveling parallel to the shore along RT. When the

boat is at point R, the captain measures the angle to the lighthouse as 35º After the

boat has traveled 2.1 miles, the captain measures the angle to the lighthouse as 70°

Answers

The exterior angle theorem states that the sum of two opposite interior  angles is equal to the measure of the exterior angle.

The distance SL= 5.77 miles between the boat and the lighthouse after travelling 2.1 miles

According to the conditions given in the problem the exterior angle is 70 degrees and one of the opposite interior angle is 35 degrees.

m∠ L + m∠R= 70°

m∠ L +  35° = 70°

m∠ L = 70°-35°

m∠ L = 35°

If one interior angle is 35 degrees the other must also be 35 degrees to make a total of 70 degrees.

The sum of all angles of the triangle is always equal to 180 degrees.

So the third angle of the triangle will be

180°= 35°+35°+m ∠S

m∠S= 110°

From the triangle

Angle theta= m∠S= 110°

Fy= SL= height

Fx= RS = base

F= RL= hypotenuse

The line SL has to be found out.

Let the Fx= 2.1 miles

Then

Fx= Fcos ∅

But Cos   ∅ =  Cos 110°= -0.342

2.1=  F (-0.342)

F= - 6.140 = hypotenuse

Now the vertical component

Fy= Fsine theta

Fy= - 6.140 sine 110°

Fy= - 6.14×0.94

Fy= -5.77 miles

The negative sign indicates that it is in the opposite direction.

https://brainly.com/question/13729598

anyone know if they can help me rq

Answers

We are given two equations, one of which has an isolated variable [tex]x[/tex].

That screams to me that substitution would be a prefered strategy here, compared to elimination, although both work.

That means we'll be substituting our value of [tex]x[/tex], which is given as [tex]-y+3[/tex], into the first equation, [tex]15x+31y=-3[/tex].

[tex]15x+31y=-3[/tex]

[tex]x=-y+3[/tex]

[tex]15(-y+3)+31y=-3[/tex]

[tex]-15y+45+31y=-3[/tex]

[tex]16y=-48[/tex]

[tex]y=-3[/tex]

With this value, we can plug it back into either of the two equations to solve for [tex]x[/tex], I'll be substituting it back into the second equation, since it's easier.

[tex]x=-y+3[/tex]

[tex]y=-3[/tex]

[tex]x=-(-3)+3[/tex]

[tex]x=6[/tex]

So our solution is [tex](6,-3)[/tex], and to check we can plug it back into the first equation.

[tex]15x+31y=-3[/tex]

[tex]15(6)+31(-3)=-3[/tex]

[tex]90-93=-3[/tex]

Which is true, so our solution is correct.

Hope this helps!

Answer:

Step-by-step explanation:

[tex]\begin{cases} 15x+31y=-3 \\\\ x=-y+3 \ \ | \times 15\end{cases} \Leftrightarrow \ominus\begin{cases} 15x+31y=-3 \\\\ 15x=-15y+45 \ \ \end{cases} \Leftrightarrow \\\\\\ 15x-15x+31y=-3-(-15y)-45 \\\\ 31y=15y-48 \\\\ 31y-15y=-48 \\\\ 16y=-48 \ \ |\div 16 \\\\ y=-3 \ \ ; \ \ x=-y+3=3+3=6 \\\\[/tex]

[tex]\huge \boldsymbol{\mathfrak {Unswer}}: x=6 \ \ ; \ \ y=-3[/tex]

Which function has a greater rate of change?​

Answers

[tex]\huge\bold{\purple{\bold{⚡EuroNow⚡}}} [/tex]

[tex]\huge\underline\mathtt\colorbox{cyan}{Its easy}[/tex]

The function for EuroNow has a greater rate of change

Work out the nth term of the following sequence:
7, 16, 31, 52, 79

its a quadratic sequence and i worked out the second difference (6) so know its 3n² but i don't know the rest

Answers

Answer:

Step-by-step explanation:

Since it is given that it is quadratic it is of the form   f(n)=an^2 + bn+c.

Since we know a few terms we can plug in to get some equations:

f(1)= a +b+c = 7

f(2)=4a+2b+c = 16

f(3)=9a+3b+c = 31

Now you've got yourself a system of three equations which I trust you can solve.

I will say that I'm not sure that this is the most efficient solution(check out the link below which might have a better solve).

Here are some nice explanations with the "real" math notation for a slightly different but related problem:

https://math.stackexchange.com/questions/2345256/how-to-find-the-nth-term-of-quadratic-sequences

Good luck!

graph using the slop and y intercept

Answers

the slope is -1/2 and the y intercept is 2

Answer:

the slop is -1/2 and they y intercept 2

HELP PLEASE I CANT FIGURE THIS OUT

Answers

I don’t understand what’s the problem?

Answer:

the slope is 1/4. the equation would be y=1/4x-7 I think

Step-by-step explanation:

how do i write it please and the answer

Answers

Use your keyboard and if that doesn’t work on the right top comet there’s a pen that should work :))

Answer:

8-9/16 or 7.4375.

Step-by-step explanation:

x=2 y=3/4

2*2*2=8

3/4*3/4=9/16

8-.5625=7.4375

Find the slope ... ...

Answers

Answer:

-2

Step-by-step explanation:

rise over run = 4/-2 = -2

How far is it around the perimeter of crater lake?

Answers

NRHP reference No. Rim Drive is a scenic highway in Crater Lake National Park in southern Oregon, United States. It is a 33-mile (53 km) loop that follows the caldera rim around Crater Lake.

A point $P$ is randomly selected from the square region with vertices at $(\pm 2, \pm 2)$. What is the probability that $P$ is within one unit of the origin

Answers

If P = (X, Y) is a point in the given square, then X and Y are i.i.d random variables each with distribution

[tex]\displaystyle P(X = x) = \begin{cases}\dfrac14 & \text{if } -2 \le x \le 2 \\ 0 & \text{otherwise}\end{cases}[/tex]

and so the joint density of X and Y is

[tex]\displaystyle P(X = x, Y = y) = \begin{cases}\dfrac1{16} & \text{if }-2 \le x \le 2 \text{ and } -2 \le y \le 2 \\ 0 &\text{otherwise}\end{cases}[/tex]

We want to find P(X² + Y² ≤ 1). Points that satisfy this inequality lie in the set

R = {(x, y) : -1 ≤ x ≤ 1 and -√(1 - x²) ≤ y ≤ √(1 - x²)}

but we can more easily describe the region in polar coordinates by setting

x = r cos(t) and y = r sin(t)

so that the set R is identical to

R' = {(r, t) : 0 ≤ r ≤ 1 and 0 ≤ t ≤ 2π}

Integrate the joint density over R' :

[tex]\displaystyle P(X^2 + Y^2 \le 1) = \iint_R \frac1{16} \, dx \, dy[/tex]

[tex]\displaystyle P(X^2 + Y^2 \le 1) = \iint_{R'} \frac r{16} \, dr \, dt[/tex]

[tex]\displaystyle P(X^2 + Y^2 \le 1) = \int_0^{2\pi} \int_0^1 \frac r{16} \, dr \, dt[/tex]

[tex]\displaystyle P(X^2 + Y^2 \le 1) = \int_0^{2\pi} \frac{1^2 - 0^2}{32} \, dt[/tex]

[tex]\displaystyle P(X^2 + Y^2 \le 1) = \frac1{32} \int_0^{2\pi} dt[/tex]

[tex]\displaystyle P(X^2 + Y^2 \le 1) = \frac{2\pi-0}{32}[/tex]

[tex]\displaystyle P(X^2 + Y^2 \le 1) = \boxed{\frac{\pi}{16}}[/tex]

to write an equation of a line that is...

parallel to 2x - 4y = 8
and goes through the point (3, -2)

Answers

Answer:

x-2y=7 or y=[tex]\frac{1}{2}x-\frac{7}{2}[/tex]

Step-by-step explanation:

Hi there!

We are given the line 2x-4y=8, and we want to write an equation of the line that is parallel to it and passes through (3, -2)

Parallel lines have the same slope, so it would be a good idea to find the slope of 2x-4y=8

The equation is currently written in standard form (ax+by=c), where a, b, and c are free integer coefficients, but a and b CANNOT be equal to 0, and a CANNOT be negative

In order to find the slope of the line, we can rewrite it in another form; for example, slope-intercept form (y=mx+b, where m is the slope and b is the y intercept)

To rewrite the line in this way, we'll need to isolate y on one side.

So start by subtracting 2x from both sides

-4y=-2x+8

Divide both sides by -4

y=[tex]\frac{1}{2}x[/tex]-2

The slope of the line 1/2, as it's in the place of where m is.

It's also the slope of the line parallel to it.

We can write the equation of the new line in slope-intercept form. Here's what we know so far:

y=[tex]\frac{1}{2}x[/tex]+b (b is a placeholder for the y intercept)

So we'll need to find b.

As the equation passes through the point (3, -2), we can use it to solve for b

Substitute 3 as x and -2 as y:

-2=1/2(3)+b

Multiply

-2=3/2+b

Subtract 3/2 from both sides:

-7/2=b

Substitute -7/2 as b in the equation:

y=[tex]\frac{1}{2}x-\frac{7}{2}[/tex]

The equation can be left as that, or you can convert it back into standard form

Subtract [tex]\frac{1}{2}x[/tex] from both sides, as the variables are on one side.

[tex]-\frac{1}{2}x[/tex]+y=[tex]-\frac{7}{2}[/tex]

Remember that the coefficient in front of x (a) CANNOT be negative, and also the free coefficients a, b, and c CANNOT be fractions.

So in order to clear the fractions and to change the signs, multiply both sides by -2

x-2y=7

Hope this helps!

Write the equation of the line that passes through the points(-6,8)and(0,-3). Put your answer in fully reduced point-slope form, unless it is a vertical or horizontal line..

Answers

We will be using the slope formula to calculate m, or the slope of the line that passes through the points (-8, -6) and (-4, 2).

Remember, when you are calculating the slope from two given points, you are finding the change in y over the change in x.

P1 = ( x1, y1 )
P2 = ( x2, y2 )

m = Δy / Δx ---> m = (y2 - y1) / (x2 - x1)


Now let's assign the given points:
(-8, -6) --> x1 = - 8 & y1 = -6
(-4, 2) --> x2 = -4 & y2 = 2


Now, let's plug these values into the slope formula:
m = (y2 - y1) / (x2 - x1)
= (2 - (-6)) / ((-4) - (-8))


Let's simplify:
(2 - (-6)) / ((-4) - (-8))
= (8) / (4)
= 2

Therefore, the slope, or m, is 2.

Answer:

y-8= -11/6 (x+6)

Step-by-step explanation:

When determining domain it is important to work from

Answers

Answer:

use graphs

Step-by-step explanation:

Another way to identify the domain and range of functions is by using graphs. Because the domain refers to the set of possible input values, the domain of a graph consists of all the input values shown on the x-axis. The range is the set of possible output values, which are shown on the y-axis.

if x and y vary inversely and x=2.5 when y =100 find x when y=25

Answers

Answer:

x is 10.

..............

If the length of each side of a cuboid decreases by 20%, find the percentage decrease in its volume.
Solution : (Identify the ratio of the length of one edge of the original cuboid and the length of one edge of the new cuboid)​

Answers

Answer:

Step-by-step explanation:

(1+25 /100) (1-20/100) (1-50/100)  <1

5/4 x 4/5 x 1/2 <1

Decrease in volume (in percent)

(1+25 /100) (1-20/100) (1-50/100)  x 100

=48.8%

Select two national treasures saved by Dolley Madison.

Gilbert Stuart's portrait of George Washington
A bust of Thomas Jefferson
The Declaration of Independence
The U.S. Constitution

Answers

Answer:

Gilbert Stuart's portrait of George Washington The U.S. Constitution

Step-by-step explanation:

Answer:  Gilbert Stuart's portrait of George Washington

The Declaration of Independence

Step-by-step explanation:

1. Mitch Hart's gross pay for this week is $425.78. He is single and claims 2 6 points

allowances. Using the percentage method of withholding, what amount

will Mitch's employer withhold from his pay for federal income tax?

Percentage Method of Withholding - Single Person

Each weekly allowance is $63.46.

Weekly Taxable The amount of income tax to

Wage

withhold is

Over But not

over

$51.00 $195.00 S0.00 plus (10% of the excess

over $51.00)

195.00 645.00 $14.40 plus (15% of the excess

over $195.00)

645.00 1482.00 $81.90 plus (25% of the excess

over $645.00)

Answers

Using the percentage method, the amount that Mitch's employer is going to withhold is $29.979

The total wage payment usually received by Mitch monthly is = $425.78Each weekly allowance is $63.46, It is noted that Mitch claims two allowances = $63.46 × 2= $126.92

Now, the amount subject to withholding from his total wage payment is;

= $(425.78 - 126.92)= $298.86

Provided that the weekly taxable wage is more than $195 but not up to $645; Then, by using the percentage method, the Federal income tax can be computed as:

= 15% of $(298.86 - 195) + $14.40= 15% of $103.86 + $14.40= $15.579 + $14.40= $29.979

Therefore, we can conclude that the amount that Mitch's employer is going to withhold is $29.979

Learn more about taxable income here:

https://brainly.com/question/17347618?referrer=searchResults

15. Describe and correct the error in finding the intercepts of the graph of the equation.
Х
6x + 9y
= 18
6x + 9(0) = 18
6x = 18
6x + Oy 18
6(0) + 9y = 18
9y =
= 18
X = 3
y = 2
The x-intercept is at (0,3), and the y-intercept is at (2, 0).

Answers

Answer:

Wrong order!

Step-by-step explanation:

There's a reason why coordinates are called an "ordered pair", and for every point (a, b) a is the x coordinate and b is the y coordinate. The calculations are correct, the way reporting them are not: the point should be (3.0) and (0,2) - in fact, all points on the x axis are of the form (p, 0) and all points on the y axis of the form (0,q)

Other Questions
Do you have 5th grade science for kids in Texas I need help with this pls help . (when u get in the document its duplicated) HELP ME OUT PLEASE!!! WILL GIVE BRAINLIEST!!! In "The Golden Tree Killer," the Haida believed the golden spruce was Carson has a smart phone data plan that costs $35 per month that includes 10 GB of data, but will charge an extra $15 per GB over the included amount. How much would Carson have to pay in a month where he used 3 GB over the limit? How much would Carson have to pay in a month where he used went over by xx GB?Total cost when over by 3 GB: Total cost when over by xx GB: 6p - 5 =133-31215 Pls help me Im being timed I really need to pass this Allie says that the expressions 4x - 2x +4 and 2(x + 2) are not equivalent because one expressionhas a term that is subtracted and the other does not. Do you agree? Explain.Yes,the expressions are equivalent. The first expression, 4x - 2x +4, can be simplified towhich equals 2(x + 2).(Use the operation symbols in the math palette as needed.) 8.6w+2.2(2w-5)=54help plz8th grade math Which part of the planet had the highest concentration of ozone in 1989? Which expression is equal to 3x/x+3+x2/x what is another word for Angle 1? After the blood collected from a heel stick is placed on the neonatal screening card, the card must dry in a horizontal position for a minimum of: What is the scale factor of the dilation of triangle PRQ?A 3B 1C 1/3D 6 Which angles are adjacent to each other? Question is located in the attached file (image) : Lp bng bin thin v v th cc hm s sau 1) (P):y=2x2 5x+3 What type of energy is best described as the energy required to break the bonds present in the reactants?Aactivation energyBpotential energykinetic energyDthermal energyhay yo who know this i need help Im not sure about this problem could anybody help me ? PROBLEM : 3andb = 7: (ab)3